Quantcast
  • Register
PhysicsOverflow is a next-generation academic platform for physicists and astronomers, including a community peer review system and a postgraduate-level discussion forum analogous to MathOverflow.

Welcome to PhysicsOverflow! PhysicsOverflow is an open platform for community peer review and graduate-level Physics discussion.

Please help promote PhysicsOverflow ads elsewhere if you like it.

News

PO is now at the Physics Department of Bielefeld University!

New printer friendly PO pages!

Migration to Bielefeld University was successful!

Please vote for this year's PhysicsOverflow ads!

Please do help out in categorising submissions. Submit a paper to PhysicsOverflow!

... see more

Tools for paper authors

Submit paper
Claim Paper Authorship

Tools for SE users

Search User
Reclaim SE Account
Request Account Merger
Nativise imported posts
Claim post (deleted users)
Import SE post

Users whose questions have been imported from Physics Stack Exchange, Theoretical Physics Stack Exchange, or any other Stack Exchange site are kindly requested to reclaim their account and not to register as a new user.

Public \(\beta\) tools

Report a bug with a feature
Request a new functionality
404 page design
Send feedback

Attributions

(propose a free ad)

Site Statistics

205 submissions , 163 unreviewed
5,047 questions , 2,200 unanswered
5,345 answers , 22,709 comments
1,470 users with positive rep
816 active unimported users
More ...

  Can we quantitatively understand quark and gluon confinement in quantum chromodynamics and the existence of a mass gap?

+ 1 like - 0 dislike
1816 views

Quantum chromodynamics, or QCD, is the theory describing the strong nuclear force. Carried by gluons, it binds quarks into particles like protons and neutrons. According to the theory, the tiny subparticles are permanently confined. A quark or a gluon cannot be taken from a proton because the strong force gets stronger with distance. How much do we understand of this phenomenon? and what do we hope to find about it by studying M-theory?

This post imported from StackExchange Physics at 2015-05-31 13:01 (UTC), posted by SE-user Joe
asked May 13, 2015 in Theoretical Physics by Joe (40 points) [ no revision ]

1 Answer

+ 2 like - 0 dislike

Confinement cannot be rigorously shown in QCD with current techniques, because all analytic results in QCD are perturbative and the perturbative expansion breaks down at low energies where the coupling becomes strong.

QCD has a negative $\beta$-function, i.e. the Yang-Mills coupling grows at lower energies and becomes weaker at high energies. But the $\beta$-function itself is only known perturbatively, that is as a power series in the coupling strength. This expansion makes sense when the coupling is weak not when it is strong. This perturbative calculation of the $\beta$-function therefore teaches us two lessons:

  1. The theory is asymptotically free, the coupling becomes weaker and weaker at high energies. Since the perturbative computation is justified in this regime, we can conclude that asymptotic freedom has been shown rigorously.
  2. At low energies the coupling becomes strong due to the $\beta$-function, but the calculation of the $\beta$-function itself relies on weak coupling. So we cannot say anything definite about this regime. However, the growing coupling is seen as a hint of confinement. One cannot, for example, compute meson masses in perturbative QCD.

Lattice computations however have been able to compute some meson masses, but they are of course numerical. Maybe somebody with more knowledge of lattice QCD can add some details.

There is the hope to study the strongly coupled regime of QCD using the holographic duality which maps QCD to a weakly coupled gravity theory (more precisely a string theory, which is closely related to M theory). There are some qualitative results, see e.g. here and here, but the holographic dual of QCD, if it exists, is not known and the quest for is has been unsuccessful.

This post imported from StackExchange Physics at 2015-05-31 13:01 (UTC), posted by SE-user physicus
answered May 13, 2015 by physicus (105 points) [ no revision ]
Why do you say "cannot be?" You should say "has not been?" For example, 3d electrodynamics is a confining theory and has been shown to be so. The proof is completely non-trivial. QCD could also be proven, but it is also completely non-trivial.

This post imported from StackExchange Physics at 2015-05-31 13:01 (UTC), posted by SE-user kηives
@kηives I agree that this was not clear enough. I meant "cannot be" shown with traditional perturbative techniques. Do you have a reference for the 3d electrodynamics proof? Is it holographic? Is the theory confining at all scales?

This post imported from StackExchange Physics at 2015-05-31 13:01 (UTC), posted by SE-user physicus
Here is the link: link.springer.com/article/10.1007/BF01961240

This post imported from StackExchange Physics at 2015-05-31 13:01 (UTC), posted by SE-user kηives
this link also has a more general proof: dx.doi.org/10.1016/0550-3213(77)90086-4

This post imported from StackExchange Physics at 2015-05-31 13:01 (UTC), posted by SE-user kηives
@kηives thanks!

This post imported from StackExchange Physics at 2015-05-31 13:01 (UTC), posted by SE-user physicus

Your answer

Please use answers only to (at least partly) answer questions. To comment, discuss, or ask for clarification, leave a comment instead.
To mask links under text, please type your text, highlight it, and click the "link" button. You can then enter your link URL.
Please consult the FAQ for as to how to format your post.
This is the answer box; if you want to write a comment instead, please use the 'add comment' button.
Live preview (may slow down editor)   Preview
Your name to display (optional):
Privacy: Your email address will only be used for sending these notifications.
Anti-spam verification:
If you are a human please identify the position of the character covered by the symbol $\varnothing$ in the following word:
p$\hbar$ysicsO$\varnothing$erflow
Then drag the red bullet below over the corresponding character of our banner. When you drop it there, the bullet changes to green (on slow internet connections after a few seconds).
Please complete the anti-spam verification




user contributions licensed under cc by-sa 3.0 with attribution required

Your rights
...